大数定律

本文中我们将研究数学期望的物理意义,以及将数学期望与其物理意义联系起来的定理:大数定律。

弱大数定律

(弱大数定律)记$(X_n)_n$为一列独立且同分布的离散随机变量。 设 $X_n$具有二阶矩。 记: \(S_n = X_1 + \cdots + X_n\) 则: \(\forall \epsilon > 0, \quad \lim_{n \to \infty} \mathbf P (|\frac{S_n}{n} - E X_1| > \epsilon) = 0\) 或称离散随机变量的算术平均值依概率收敛至其期望。

弱大数定律实际上对于不具有二阶矩的随机变量也成立,但是其证明更加复杂,因此我们只证明更容易的情况。 这个定律对两两独立的变量也成立,对非离散的随机变量也成立,但我们只研究离散情况。

不失一般性地,我们假设这一列随机变量的期望为零。 设$\delta > 0$,我们要证: \(\mathbf P(|\frac{S_n}{n}| \ge \delta) \xrightarrow{n \to \infty} 0\) 写成ε-δ语言的形式,即对于任何$\varepsilon > 0$,都有: \(\exists N, \; \forall n \ge N, \quad \mathbf P(|\frac{S_n}{n}| \ge \delta) < \varepsilon\) 证明的思路为利用一个实数$M$将随机变量二分为大于$M$和小于等于$M$两个部分,然后分别计算其上界,最后根据$\varepsilon$取$M$并构造$N$即可。
设: \(Y_n = X_n \cdot \mathbb{1}_{|X_n| \le M}, \quad M \in \mathrm R_+\) 那么$(Y_n)$也是一列独立同分布的离散随机变量。 设$Z_n = X_n - Y_n$,那么$(Z_n)$也是独立同分布的。 现在记 \(\overline{X_n} = \frac{1}{n} \sum_{i=1}^n X_i\) 同理我们也能定义$\overline{Y_n}, \overline{Z_n}$,那么我们有: \(\begin{aligned} \overline{X_n} &= \overline{X_n} - \underbrace{\mathbf E \overline{X_n}}_{= 0} \\ &= \overline{Y_n} + \overline{Z_n} - (\mathbf E \overline{Y_n} + \mathbf E \overline{Z_n}) \\ &= (\overline{Y_n} - \mathbf E \overline{Y_n}) + (\overline{Z_n} - \mathbf E \overline{Z_n}) \\ \implies | \overline{X_n} | &\le |\overline{Y_n} - \mathbf E \overline{Y_n}| + |\overline{Z_n} - \mathbf E \overline{Z_n}| \\ \implies \{ |\overline{X_n}| \ge \delta \} &\subset \{ |\overline{Y_n} - \mathbf E \overline{Y_n}| \ge \delta/2 \} \cup \{ |\overline{Z_n} - \mathbf E \overline{Z_n}| \ge \delta/2 \} \end{aligned}\) 最后一个包含关系可以由反证法验证。
现在我们来分别计算两个部分的上界。 首先对第一个部分,注意到 \(\forall n, | Y_n | \le M \implies \mathbf V(Y_n) \le \mathbf E (Y_n)^2 \le M^2\) 使用切比雪夫不等式: \(\begin{aligned} \mathbf P (|\overline{Y_n} - \mathbf E \overline{Y_n}| \ge \delta/2) & \le \frac{\mathbf V(\overline{Y_n})}{(\delta/2)^2} = \frac{4 \mathbf V(\overline{Y_n})}{\delta} \\ & = \frac{4}{n^2 \delta} \mathbf V(\sum_{i=1}^n Y_n) \\ & \le \frac{4 M^2}{n \delta^2} \end{aligned}\) 对第二个部分,使用马尔可夫不等式: \(\begin{aligned} \mathbf P (\overline{Z_n} - \mathbf E \overline{Z_n}| \ge \delta/2) & \le \frac{2 \mathbf E|\overline{Z_n} - \mathbf E \overline{Z_n}|}{\delta} \le \frac{4 \mathbf E| \overline{Z_n} |}{\delta} \\ & \le \frac{4}{n\delta} \mathbf E(\sum_{i=1}^n |Z_n|) = \frac{4}{\delta} \mathbf E(|Z_n|) \end{aligned}\) 现在我们需计算$\vert Z_n \vert$的期望: \(\mathbf E |Z_n| = \mathbf E |Z_1| = \mathbf E(|X_1| \mathbb 1_{|X_1| > M}) = \sum_{x > M} |x| \mathbf P(X_1 = x)\) 我们知道$|X_1|$具有有限的期望,且: \(\begin{aligned} \mathbf E |X_1| &= \sum_{x \in X_1(\Omega)} |x| \mathbf P(X_1 = x) \\ &= \sum_{x \le M} |x| \mathbf P(X_1 = x) + \sum_{x > M} |x| \mathbf P(X_1 = x) \\ &= \lim_{M \to \infty} \sum_{x \le M} |x| \mathbf P(X_1 = x) \end{aligned}\) 从而当$M$趋于无穷大时,$|Z_n|$的期望可以任意小,这意味着我们可以通过选择足够大的$M$来限制$\mathbf E(|Z_n|)$,从而限制第二个部分的上界。 现在,我们取$M$,使得$\mathbf E |Z_n| < \frac{\delta \varepsilon}{8}$,代入第二个部分,可得: \(\mathbf P (|\overline{Z_n} - \mathbf E \overline{Z_n}| \ge \delta/2) \le \frac{\varepsilon}{2}\) 从而完成了第二个部分的限制。 对第一个部分,注意到不等式中存在$n$,因此我们可以通过选择$N$来完成第一个部分的限制。 取 \(N = \left\lfloor \frac{8 M^2}{\varepsilon \delta^2} \right\rfloor + 1\) 那么: \(\forall n \ge N, \; \mathbf P (|\overline{Y_n} - \mathbf E \overline{Y_n}| \ge \delta/2) \le \frac{\varepsilon}{2}\) 从而完成了对第一个部分的限制,这就完成了这一定理的证明。

依概率收敛和几乎必然收敛

依概率和几乎必然收敛的定义

设$(X_n)$为一列实值离散随机变量,记$C$: \(C = \{ \omega \in \Omega \; | \; \lim_{n \to \infty} X_n(\omega) = 0 \} = \{ \lim_{n \to \infty} X_n = 0 \}\) 则$C$是一个良定义的事件。

把极限写成ε-δ语言的形式: \(\omega \in C \iff \forall k \ge 1, \; \exists n \ge 0, \; \forall p \ge n, \quad |X_p(\omega)| < \frac{1}{k}\) 注意到这里我们不使用$\varepsilon$,这是因为实数域是不可数的,这会为我们的证明带来困难。 我们使用实数的阿基米德性质来用一个自然数限制$|X_p(\omega)|$。 从而: \(C = \bigcap_{k \ge 1} \bigcup_{n \ge 0} \bigcap_{p \ge n} \{ \omega \in \Omega \;|\; |X_p(\omega)| < \frac{1}{k} \}\) 由于σ-代数关于可数个并和交封闭,因此$C$是一个事件。

称一列离散随机变量$(X_n)$几乎必然收敛(to converge almost surely)至零,若: \(\mathbf P ( \lim_{n \to \infty} X_n = 0) = 1\) 简记为: \(X_n \xrightarrow{\text{a.s.}} 0\) 称其依概率收敛(to converge in probability)至零,若: \(\forall \varepsilon > 0, \; \lim_{n \to \infty} \mathbf P(|X_n| > \varepsilon) = 0\) 简记为: \(X_n \xrightarrow{\mathbf P} 0\)

我们已经说明了$\{ \lim_{n \to \infty} X_n = 0 \}$是一个事件,因此这些概念都是良定义的。

两种收敛的关系

若一列离散随机变量几乎必然收敛至零,那么它依概率收敛至零。

该列变量几乎必然收敛至零,则: \(\mathbf P (C) = 1 \iff \mathbf P(C^\complement) = 0\) 又: \(C^\complement = \bigcup_{k = 1} \bigcap_{n > 0} \bigcup_{p \ge n} \{ |X_p| \ge \frac{1}{k} \}\) 从而 \(\mathbf P (\bigcap_{n > 0} \bigcup_{p \ge n} \{ |X_p| \ge \frac{1}{k} \}) = 0, \; \forall k \ge 1\) 不难注意到$(\cup_{p \ge n} \{ \vert X_p \vert \ge \frac{1}{k} \})_n$是一列单减的事件,这是因为随着$n$增加,被并起来的集合越来越少,因此我们可以使用概率的单调连续性: \(\begin{aligned} 0 = \mathbf P (\bigcap_{n > 0} \bigcup_{p \ge n} \{ |X_p| \ge \frac{1}{k} \}) &= \lim_{n \to \infty} \mathbf P (\bigcup_{p \ge n} \{ |X_p| \ge \frac{1}{k} \}) \\ &\ge \lim_{n \to \infty} \mathbf P(|X_n| \ge \frac{1}{k}) = 0 \end{aligned}\) 从而: \(\forall k \ge 1, \quad \lim_{n \to \infty} \mathbf P(|X_n| \ge \frac{1}{k}) = 0\) 现在,利用实数的阿基米德性质,取$k = \lfloor \frac{1}{\varepsilon} \rfloor$,即有: \(\{ |X_n| \ge \varepsilon \} \subset \{ |X_n| \ge \frac{1}{k} \}\) 从而 \(\mathbf P(|X_n| \ge \varepsilon) \xrightarrow{n \to \infty} 0\)

若一列离散随机变量满足 \(\forall \varepsilon > 0, \; \sum \mathbf P(|X_n| > \varepsilon) < \infty\) 则其几乎必然收敛至零。

这里给出一个证明的框架。 首先,由博雷尔-坎特利第一引理,可知,若 \(\forall \varepsilon > 0, \; \sum \mathbf P(|X_n| > \varepsilon) < \infty\) 则: \(\forall \varepsilon > 0, \mathbf P (\limsup_{n \to \infty} \{ |X_n| > \varepsilon \}) = 0\) 即: \(\forall \varepsilon > 0, \mathbf P (\bigcap_{n>0} \bigcup_{p \ge n} \{ |X_n| > \varepsilon \}) = 0\) 现在用$k$替换$\varepsilon$,可得: \(\forall k \ge 1, \mathbf P (\bigcap_{n>0} \bigcup_{p \ge n} \{ |X_n| \ge \frac{1}{k} \}) = 0\) 从而: \(\mathbf P(\bigcup_{k \ge 1} \bigcap_{n>0} \bigcup_{p \ge n} \{ |X_n| \ge \frac{1}{k} \}) \le \sum_{k = 1}^\infty \mathbf P (\bigcap_{n>0} \bigcup_{p \ge n} \{ |X_n| \ge \frac{1}{k} \}) = 0\) 取补集,即可得: \(\mathbf P(\bigcap_{k \ge 1} \bigcup_{n>0} \bigcap_{p \ge n} \{ |X_n| < \frac{1}{k} \}) = 1\) 即$\mathbf P(C) = 1$。

设$(\varepsilon_n)_{n \in \mathbb N}$为一列收敛至零的实数列,且一列离散随机变量$(X_n)$满足: \(\sum_{n \in \mathbb N} \mathbf P(|X_n| \ge \varepsilon_n) < \infty\) 那么$X_n$几乎必然收敛至零。

这个命题意味着,若我们能够将$\vert X_n \vert$限制地足够小,那么其就能几乎必然收敛至零。

强大数定律

强大数定律的特殊情况

我们首先证明一个比较容易的情况。

设$(X_n)$为一列独立同分布的离散随机变量,且: \(\exists K \ge 0, \; \forall n, \; \mathbf E X_n^4 \le K\) 设: \(S_n = (X_1 + \cdots + X_n)\) 则$\frac{S_n}{n} - \mathbf E X$几乎必然收敛至零。

首先,我们只研究期望为零的随机变量,设$X^\prime_n = X_n - \mathbf E X_n$。 我们马上证明这个变量满足题设。 首先不难验证这一列变量也是独立同分布的,因为其只是为每个变量减去一个常数。 然后,我们验证其四次方有界: \(\begin{aligned} \mathbf E(X_n^\prime)^4 &\le \mathbf E(|X_n| + | \mathbf E X_n|)^4 \\ &\le 2^4 \mathbf E \max (|X_n|, |\mathbf E X_n|)^4 \\ &\le 2^4 \mathbf E (X_n^4 + (\mathbf E X_n)^4) \\ &\le 2^4 (K^4 + (\mathbf E X)^4) \le K^\prime \end{aligned}\) 接下来我们只需要求出$S_n$的上界即可。 \(\mathbf E(S_n^4) = \mathbf E ((X_1 + \cdots + X_n)^4) = \sum_{i,j,k,l} \mathbf E(X_i X_j X_k X_l)\) 设$i \not \in \{j,k,l\}$,那么由独立性,可知: \(\mathbf E (X_i X_j X_k X_l) = \mathbf E X_i \mathbf E(X_j X_k X_l) = 0\) 从而这个乘法展开后只有两种情况: \(\mathbf E(S_n^4) = \sum_{1\le i \le n} \mathbf E(X_i^4) + \binom{4}{2} \sum_{1 \le i < j \le n} \mathbf E(X_i^2 X_j^2)\) 利用琴生不等式(或方差非负): \(E(X_i^4) \ge (\mathbf E (X_i^2))^2 \implies \mathbf E(X_i^2) \le \sqrt{\mathbf E(X_i^4)} \le \sqrt{K}\) 从而: \(\mathbf E(X_i^2 X_j^2) \le K\) 原等式可放缩为: \(\mathbf E(S_n^4) \le n K + 3 n(n-1) K \le 3 n^2 K\) 利用马尔可夫不等式: \(\mathbf P(|\frac{S_n}{n}| \ge \varepsilon) = \mathbf P (|\frac{S_n}{n}|^4 \ge \varepsilon^4) \le \frac{\mathbf E(\frac{S_n}{n})^4}{\varepsilon^4} \le \frac{3K}{n^2 \varepsilon^4}\) 从而$\sum \mathbf P(|\frac{S_n}{n}| \ge \varepsilon) < +\infty$,因此其几乎一定收敛至零。

强大数定律

(强大数定律)设$(X_n)$为一列两两独立的同分布离散随机变量,且具有二阶矩,那么 \(\frac{S_n}{n} - \mathbf E X \xrightarrow{\text{a.s.}} 0\)

和弱大数定律一样,强大数定律并不要求具有二阶矩,这里加入这个条件只是为了降低证明难度。

我们知道,一个实值函数可以写成其正部分和负部分之和: \(X_n = X_n^+ - X_n^-, \; |X_n| = X_n^+ + X_n^-\) 注意到$X_n^+$和$X_n^-$都是满足题设的正随机变量,我们可以只对正随机变量证明这个命题,因此我们设$X_n \ge 0$。 设$\varepsilon > 0$,构造函数: \(\varphi(n) = \lfloor (1+\varepsilon)^n \rfloor + 1\) 满足: \((1+\varepsilon)^n \le \varphi(n) \le (1+\varepsilon)^n + 1\) 利用切比雪夫不等式: \(\begin{aligned} \mathbf P \left( \left|\frac{S_{\varphi(n)}}{\varphi(n)} - \mathbf E X_1 \right| \ge \frac{1}{(1+\varepsilon)^\frac{n}{4}} \right) &\le \mathbf V \left(\frac{S_{\varphi(n)}}{\varphi(n)}\right) (1+\varepsilon)^{n/2} \\ &= \mathbf V(S_{\varphi(n)}) \frac{(1+\varepsilon)^{n/2}}{\varphi^2(n)} \\ &= \sum_{i=1}^{\varphi(n)} \mathbf V(X_i) \frac{(1+\varepsilon)^{n/2}}{\varphi^2(n)} \\ &= \mathbf V(X_1) \frac{(1+\varepsilon)^{n/2}}{\varphi(n)} \\ &\le \mathbf V(X_1) (1+\varepsilon)^{-n/2} \end{aligned}\) 从而: \(\sum \mathbf P \left( \left|\frac{S_{\varphi(n)}}{\varphi(n)} - \mathbf E X_1 \right| \ge \frac{1}{(1+\varepsilon)^\frac{n}{4}} \right) \le +\infty\) 从而: \(\frac{S_{\varphi(n)}}{\varphi(n)} - \mathbf E X_1 \xrightarrow{\text{a.s.}} 0\) 现在,我们注意到$\forall k \in [\varphi(n), \varphi(n+1)]$,都有: \(\frac{S_\varphi(n)}{\varphi(n+1)} \le \frac{S_k}{k} \le \frac{S_\varphi(n+1)}{\varphi(n)}\) 简单的变换之后,我们可以用$\frac{S_\varphi(n)}{\varphi(n)}$和$\frac{S_\varphi(n+1)}{\varphi(n+1)}$限制$\frac{S_k}{k}$,从而使其也满足这个不等式,因此也几乎必然收敛。 这就完成了对原命题的证明。

更新时间: